Find Point c that satisfies the Mean Value Theorem

Click For Summary
To find the point "c" that satisfies the Mean Value Theorem for the function f(x) = (x-1)/(x+1) on the interval [4,5], the derivative f'(x) = 2/(x+1)^2 was calculated. The values f(4) and f(5) were determined to be 3/5 and 2/3, respectively. Setting the derivative equal to the average rate of change, the equation 2/(x+1)^2 = 1/15 was formed. The quadratic equation derived from solving this was 0 = c^2 + 2c - 29, leading to potential solutions of c = 3.475 and c = -7.475. However, the correct point c that satisfies the theorem is approximately 4.48, indicating a need for careful calculation in the process.
Schaus
Messages
118
Reaction score
5

Homework Statement


Find the point "c" that satisfies the Mean Value Theorem For Derivatives for the function
## f(x) = \frac {x-1} {x+1}## on the interval [4,5].
Answer - c = 4.48

Homework Equations


##x = \frac {-b \pm \sqrt{b^2 -4ac}} {2a}##
##f'(c) = \frac { f(b) - f(a)} {b-a}##

The Attempt at a Solution


I found the derivative
##f'(x) = \frac {2} {(x+1)^2}##
a) ## f(4) = \frac {4-1} {4+1} = \frac {3} {5}##
b) ## f(5) = \frac {5-1} {5+1} = \frac {2} {3}##
Substituting in my values
## \frac {2} {(x+1)^2} = \frac { \frac {2} {3} - \frac {3} {5}} {5-4}##
## \frac {2} {(x+1)^2} = \frac {1} {15}##
## 30 = c^2 +2c +1##
## 0 = c^2 +2c -29##
##x = \frac {-2 \pm \sqrt{2^2 -(4)(1)(-29)}} {2(1)}##
Which gives me 3.475 and -7.475. I'm not sure where I went wrong, any help would be greatly appreciated. Neither one of there are in my interval and my solution says it needs to be 4.48.
 
Physics news on Phys.org
look the equation:
$$x = \frac {-2 \pm \sqrt{2^2 -(4)(1)(-29)}} {2(1)}$$
and solve it carefully
 
I swear I put that in my calculator a few times but this last time it worked... Thanks a lot anyways :)
 
Question: A clock's minute hand has length 4 and its hour hand has length 3. What is the distance between the tips at the moment when it is increasing most rapidly?(Putnam Exam Question) Answer: Making assumption that both the hands moves at constant angular velocities, the answer is ## \sqrt{7} .## But don't you think this assumption is somewhat doubtful and wrong?

Similar threads

  • · Replies 4 ·
Replies
4
Views
1K
  • · Replies 12 ·
Replies
12
Views
1K
Replies
3
Views
2K
  • · Replies 10 ·
Replies
10
Views
2K
  • · Replies 11 ·
Replies
11
Views
2K
  • · Replies 10 ·
Replies
10
Views
2K
  • · Replies 21 ·
Replies
21
Views
2K
  • · Replies 2 ·
Replies
2
Views
2K
Replies
4
Views
2K
  • · Replies 8 ·
Replies
8
Views
2K